Davis Relativity Model (Debate/discussion edition)

  • 158 Replies
  • 32320 Views
*

AltSpace

  • Flat Earth Believer
  • 411
  • Neo-Planarist
Davis Relativity Model (Debate/discussion edition)
« on: September 24, 2017, 08:06:52 PM »
I decided to write up an explanation for the model I accept, in regards to the nature of the Earth. I have seen it done before and I figured that since the Flat Earth model I accept is unique among the others and not commonly held by other planarists. I will outline it here, and take criticisms, edits, and corrections from any feedback.
If I am to put a name on this model, it would be most accurate to deem it the “Davis Relativity Model”, named after John Davis, the American Flat Earth Society Secretary (shout-out to Davis, you are awesome) for putting this idea out, and it heavily relies on relativity. Links I recommend for some explanation of it: https://www.theflatearthsociety.org/tiki/tiki-index.php?page=Davis+Model
https://www.theflatearthsociety.org/home/index.php/blog/einsteins-relativity-proves-earth-flat
It is important to note that my way of explaining it will not be like Davis may describe it, in fact, I may track off of what the model was originally perceived to be, but this is how I look at it, and interpret it. 
This post will be written in the following order:
1.   Basic Explanation
2.   The pillars of this model
3.   Why it should be more considered in mainstream FE
4.   FAQ
I call it The Davis Relativity Model because:
1. John Davis put this out as a flat earth model originally.
2. It heavily relies on Relativity, General relativity specifically. It is like "The Relativity Universe" as the term "The magnetic Universe" is to the Magnetic Flat Earth theory.

Links I recommend for some further explanation of it: https://www.theflatearthsociety.org/tiki/tiki-index.php?page=Davis+Model

https://www.theflatearthsociety.org/home/index.php/blog/einsteins-relativity-proves-earth-flat


The Earth is a flat plane including the sun, moon, planets, and stars. The Earth is no more unique in terms of its physical geometry than the other celestial bodies. The universe consists of aether (basically space-time), which is bent around any large mass. Think of any mass as ‘displacing’ the aether, a large mass will bend aether around it. All large masses will bend aether around them in a similar way, following that distortion of otherwise “flat” space. This means that the Earth bends aether around it, including the moon, sun, planets, and distant stars.
Aether is a term that represents the fabric of the 4-dimensional space-time continuum, in which times and distances between event pairs vary by the inertial frame of reference in which they are determined, while any event pair remains independent of the inertial frame of reference in which they are recorded. Aether can bend due to energy/mass, which is basically described in Einstein’s field equations, basically put out as:
Gμν=8πTμν
Where Gμν is the Einstein Tensor (with the geometry of space-time), and Tμν is the stress-energy tensor, which describes the movement of matter and energy through aether.
Please note that this is how aether is defined into this specific flat earth model, and is not to be taken into the context of other models.
Now, it is important to define “flat plane” in this model because this model does fall under the category of being a flat earth model. Flat in this case, does not mean two-dimensional, anyone could agree that the existence of mountains is not something that defines the geometry of the Earth when we are talking “flat” or “round” Earth. Rather, it is something based on space and how it relates to the Earth. A flat plane would be defined by the ability to traverse it in a straight line between two spatial coordinates. A “straight line” would be a line in a constant direction in three dimensional space, or any tangent vector on the surface always touching across it would be a flat surface and therefore a flat earth. So, if the Earth is able to be traversed in a straight line, it follows under the definition of a flat plane. Due to the fact that space is bent around the Earth, a straight line traversing the Earth in bent space will appear to curve relative to an observer in flat space around it, while maintaining a straight line since space curves independent of this line. This means that it maintains itself as straight but in simplistic terms, ‘space curves instead of the line’. With this, we start jumping into Frames of reference, which is an important concept in relativity. In curved aether, all frames are non-inertial, with it having acceleration in respect to an inertial reference frame. A great analogy is an elevator in space accelerating in a direction at 9.81 m/s/s (accelerating by going 9.8 meters per second faster for every second that goes by), which is indistinguishable from standing on Earth.
We can deduce that in the Earth’s non-inertial reference frame, the bending of space makes travelling through it as if in a straight line through flat space, but the space distortion around it changes its direction from a frame of reference independent of this bent space, so, we can define the path of an object through this linear direction as straight, and therefore flat. Flat can also and accurately be defined as the straight path of an object according to Newton's first law, which would be considered a satellite. Such a path defined as parallel to a straight path through space defines the path as flat, and therefore the Earth as flat. If we were to leave the Earth and look back at it, it would indeed appear as a curved spheroid, this is because the aether bends around it and therefore it becomes apparent when you can see across it, but from this appearance, we certainly can’t distinguish whether it is flat or literally round as we cannot perceive how space is being affected from your standpoint away from Earth (The Ferrari Effect). The same applies to the sun, moon, and planets, they are perceived as spheroids from our standpoint on Earth, but you couldn’t leap to the conclusion that it is surrounded by flat space from visual appearance. Due to the fact that aether bends due to mass like objects displace mediums, the Earth is flat and so are other masses throughout the universe.

Another important part of the model here is geocentricity. Flat Earth models are generally geocentric. In this model, the universe is considered as geocentric as well. In relativity, only the motion of two material bodies relative to each other can be physically detected, but the motions taking place are not absolute, they are relative. So, relative to our frame of reference, geocentrism is equally valid to heliocentrism since either can explain the relative motions, to say one is false and the other true implies that the physical motions taking place are absolute, but they are in fact not absolute according to relativity. In fact, on Earth, we take geocentrism as a valid framework since we are observing bodies in motion around us. Telescopes track and move with celestial bodies, it is just as if it is us as stationary and the celestial bodies rotating around us. It turns out to be not only a valid framework to assume geocentrism, but also a more relevant one, since us as observers are stationary relative to the Earth. This ties back into the elevator vs gravitational field example. Imagine that we are in a closed room with Mass m suspended on a spring. We suddenly observe the spring expand. What is the cause of this? We can give two explanations, either, the closed room has received an upward acceleration which the inertia of m giving a downward pull that is opposite of the direction of acceleration, or, a gravitational field that is directed downward has arisen, or at least a greater pull. It is impossible to distinguish between the two given that they cannot see outside of this closed room. Same thing applies to throwing an object up, in an accelerating elevator, the floor would accelerate up to meet the object. In a stationary room in a gravitational field, gravity would pull the object down relative to the stationary gravitating mass that "turned on". They are essentially equivalent, and so sometimes the phrase "ground accelerates up to meet the mass" is used as a representation of an object falling. Now, how is this relevant? The relative mechanics involved here give out an equivalence, a rotating heliocentric Earth and a stationary geocentric Earth under Machian mechanics. Under relativity, there are no "absolute" rotations, it is relative to the co-ordinates you choose as a frame of reference. The Earth is the most basic one for us as we are observers on Earth.
What about Foucault’s pendulum, the Coriolis Effect, and the eotvos effect? According to Mach’s principle (by Ernst Mach), the local reference frame has a direct relation to the movement of the celestial bodies around us. This would mean that the effect of the rotation of celestial bodies around Earth influencing our local reference frame is indistinguishable from the rotation of the Earth itself. A great analogy is a bucket of water. Imagine we have a stationary bucket of water; they are both stationary relative to each other and an independent observer. Now, if I stir the water, it will rotate relative to the stationary bucket from our point of view. However, if we were to instead rotate the bucket relative to the stationary water, the water would begin to move with the bucket, with the water and bucket both rotating from our observation, so the rotation of the bucket produced the same effect as the rotating water relative to the stationary bucket. This analogy applies to Mach’s principle; the rotation of the celestial bodies relative to the stationary earth would be able to produce the same effects as a rotating earth due to the pull of the celestial bodies. These would pull the pendulum along and cause the Coriolis Effect due to the change in rotation speed of the stars in the celestial sphere with latitude, since it is an inertial force. The pull of the stars would give a greater centrifugal force around the area of the pull of largest perimeter of the celestial sphere, the equator. Remember, from our framework of geocentrism, the rotation of the stars is assumed rather than the Earth, so they are indeed rotating like the Earth would be accepted to be like, with them rotating on the Polaris and Octantis axis of rotation. The North Star would be nearly stationary. Here is a very informative paper on the topic of Mach’s principle:http://www.commonsensescience.org/pdf/articles/machs_principle_and_the_concept_of_mass_fos_v16n3.pdf
Now, how are the celestial bodies “pulling” along the Earth in this model? That would be the bending of the aether with the celestial bodies rotating around earth, also may be known as frame-dragging. As the celestial bodies rotate around Earth, they bend aether and move around Earth which directly affects our local frame of reference by distorting it slightly,which gives it a “pull” by a change in space geometry. The aether bend keeps us to Earth, acting as acceleration, like the elevator example. It is essentially a non-inertial frame of reference. The bent aether is equivalent to acceleration and flat aether is equivalent to freefall. Think of a bowling ball on a trampoline, all balls near it will roll towards it, or circle around it given a rolling motion to circle it. Larger masses will bend aether more, making it “steeper”, so acceleration is increased.

The Pillars of this Model
This model has a few main supports that keep it standing; crippling any of the supports will damage it.
The Pillars:
1.   Theory of General Relativity
2.   Mach’s principle
3.   Aether (not the luminiferous aether medium of the late 19th century, but rather a term for space-time, which is influenced by mass)
4.   The Ferrari Effect

The Ferrari Effect is something not so well known outside of the Flat Earth Society. The Ferrari Effect is basically the effect of viewing the Earth and it appearing round (spherical) due to curved space. The appearance is actually an illusion due to how we interpret space in our minds, when viewing space as curved, we can’t distinguish it from a round Earth at a glance, just like the accelerating elevator and standing on Earth with the gravity to give it that acceleration. The Earth’s geometry following the curved space is what makes it flat, so we can essentially say that areas of high density in molecular clouds collapsing into stars is them “flattening out”.

I defined flat Earth as the Earth being able to traverse it in a three dimensional straight line. Just like the path of a object in accordance with Newton's first law at a specific velocity.
This, on earth, would be defined as drawing a straight line from the apex of a parabolic path of a ball, forming a tangent. If it could be demonstrated that Earth can be traversed between two spatial coordinates in a straight path across, it satisfies a flat Earth.
Now, how would we demonstrate this? We can determine the nature of space's relation to Earth.
In the early 20th century, Albert Einstein proposed his theory of general relativity where space is non Euclidean and is the equivalent of acceleration. Standing on Earth and an elevator accelerating at 9.81 m/s/s. This means acceleration would be indistinguishable from a gravitational field. Space curves and therefore affects the straight path of any object. If this is the case, and space bends around any object, as long as the acceleration across it is relatively constant like is the case on Earth. This is because the change in bend of space gives the acceleration and keeps things on Earth. So, from this, we can deduce that a straight line follows the bend if space, giving a flat Earth.

Now, from an outer observer, it appears as if a straight line is curved since it follows the bend in space, but the observer following the bend is following a straight line while space is bending their path relative to outer flat space.
This is the Ferrari Effect, a prediction by the philosopher and free thinker Leo Ferrari.

Now, how do we verify this prediction? Has it been verified as an accurate model?
Yes, it has. By observing distant objects in the universe, the path of light, and the deflection of radio waves near large masses. The Ferrari Effect relies on the conception of space by general relativity.
It has been experimentally verified and observed that the path of light through space deflects relative to us as predicted by the curvature of space-time.
Here's a link with basic description and sources: http://w.astro.berkeley.edu/~jcohn/lens.html
As you can see in the diagram presented, the straight path of light follows the curvature of space, space defines the path of an object and so a straight line in curved space implies flatness.
Now what is the angle of deflection as described mathematically with this phenomena and light?
The angle of deflection = 4GM/rc^2
Where G is the gravitational constant, M is the mass, r is the distance from the mass, and c is the speed of light in a vacuum.
According to a study done with telescopes observing radio waves bear the sun, the deflection of radio waves by the sun precisely, and it confirmed the general relativity prediction of bent space time to a high degree (within 0.03 %), here it is as published in the Astrophysical journal:https://arxiv.org/abs/0904.3992
This lensing effect has been observed with distant galaxies with long red shifts and the sun. Another identification published:https://arxiv.org/abs/1405.3661
This effect has been observed with solar eclipses and visible stars bear the sun, verifying the predictions of general relativity.www.google.com/amp/s/www.wired.com/2009/05/dayintech_0529/amp/
Also, time is affected too by this curvature, which makes it so atomic clocks on Earth run slightly slower than farther away from it, the Hafelle-Keating experiment confirmed this by  comparing clocks of planes flying east and west and a stationary clock on the Earth's surface and found an inconsistency. These clocks were cesium beam atomic clocks. Here's where you can obtain the published paper on it:http://science.sciencemag.org/content/177/4044/166
This is a confirmation of general relativity and lacks an explanation by Newtonian gravitation (Basic Round Earth explanation with Minkowski space to explain the Earth's geometry).

 If as general relativity claims, acceleration on Earth (accelerating free fall) is the result of the bending of space-time, then the curve in time and space accelerates any object to it. Think of a stationary object moving through time even though it is stationary in three dimensional space. Time is curved and so its path is curved. This curve is like a parabola on a graph. So, this object accelerates towards the Earth. It does this across Earth almost consistently, this consistent curve in space-time has a straight line between spatial coordinates travel a straight path through space while appearing to travel a non-Euclidean path from an independent frame of reference.
“Two things are infinite: the universe and human stupidity; and I'm not sure about the universe.”
― Albert Einstein

*

AltSpace

  • Flat Earth Believer
  • 411
  • Neo-Planarist
Re: Davis Relativity Model (Debate/discussion edition)
« Reply #1 on: September 24, 2017, 08:08:09 PM »
The Ferrari Effect
The Ferrari Effect is the effect of visualizing a flat plane as being a spheroid due to the curvature of aether (space-time dimensions). This was a prediction made by the philosopher Leo Ferrari that is currently a major concept in the Davis Relativity Model. Despite common conceptions, the appearance of a spherical Earth is not what defined a flat vs round surface geometry. A “flat earth” is defined by its surface geometry being so that it is traversable in a straight line as defined by the direction of linear motion according to Newton’s first law. With non-Euclidean space, a flat earth appears indistinguishable from a spheroid earth from an external point of view (just like acceleration and gravitation), what separates them is the geometry of space and its relation to Earth’s surface. Since space defines direction and geometry, the deformation of space relative to homogenous space has an equivalent definition of the geometry of a mass; it is just defined in relation to how space is changing or non-homogenous. This is based on general relativity and its conception of space.
The Ferrari Effect deals with the visual appearance of Earth, and its main implications include the possibility of orbit (considering the non-Euclidean nature of space), the potential accuracy space travel as presented by space agencies/NASA today, and an explanation of apparent curvature at high altitudes. The Ferrari Effect allows for the consistency between flat earth and modern space travel, since the apparent spheroid shape of the Earth from space travel footage and photos. Typical Flat Earth models and/or representations include the assumption that space travel is faked as a conspiracy, this is largely because of the inconsistency of the claims and apparent documentation of space travel and flat earth models and theories. However, The Ferrari Effect predicts this phenomenon and so is consistent with the documentation of modern space travel.

The Ferrari Effect was verified by experimentation with general relativity and the observations of space travel.
Why it should be more considered in mainstream FE
As anyone with a fair understanding in “Flat Earth 101” (the basics of flat earth views and models) knows, this specific model is very unique and significantly off of the mainstream. This model differs in numerous ways to the classical mainstream ones:
1.   There is no accurate map projection that conveys the surface, since space is non-Euclidean in this model and therefore can’t be mapped on 2D to complete scale.
2.   There is no surrounding ice wall; this model is bi-polar (two poles, north and south), allowing north-south circumnavigation.
3.   There is no ‘edge’ or surrounding worlds beyond; this model has the Earth contained in non-Euclidean space, so the curved space meets around it.
4.   The celestial bodies are not and need not be close to Earth, space curves away relative to the outer bodies surrounding straight across flat space.
5.   Outer space as conceived by modern society does exist, in contrast to the mainstream FE view that the celestial bodies are in a near, small, and confined space above us.
6.   Satellites, space travel, Apollo moon landings, astronauts, and modern space exploration are all consistent with this model, as opposed to the mainstream FE concept of “The Conspiracy” with the faking of space travel.
7.   This FE model is most consistent with modern mainstream science and with general relativity.
Why should planarists consider this? It appeals to skeptical globe earth advocates since it describes and works in terms of relativity. Any skeptic approaching flat earth will have many objections, questions, and potential ridiculing to go through. This will give the skeptic a reason to rethink how they originally thought this debate really means. Not only that, it is a model that can be held up to scrutiny, any flat earth model with that characteristic deserves to be considered. It also gets us to ask how we should draw the line between flat vs round earth. I consider this to be among the most advanced FE models today.
FAQ
How do basic phenomena like day-night and seasons work in this model?
 Day-night occurs by the sun moving relative to the Earth in a circle around it, and the curved aether gives it its apparent position and the sun sets to night when the sun is facing away from your position relative to the aether bend you are in. Seasons are the varying position of the sun relative to the Earth in a annual cycle, with its position directly perpendicular to the tropic of cancer in the June solstice, the equator at the equinox, and the tropic of Capricorn in the December solstice.
How does Gravity work?
Masses will follow the aether bend around and towards the Earth’s large mass; this gives acceleration and therefore makes an observer standing on Earth in a non-inertial reference frame. The change in the aether ‘fabric’ will accelerate us to Earth. The bend in space and time will accelerate the path of any object to Earth. Think of it as an 'acceleration vector', which orbit mains a perpendicular position to.
But isn’t it true that the acceleration across Earth is not completely uniform and so it’s invalid to compare it to a uniformly accelerating elevator?
For the purposes in mind here with that scenario, this is not important and an irrelevant consideration, since the acceleration experienced by a single observer standing somewhere on Earth compared to an accelerating elevator is what we are imagining.
This model seems very similar to the round earth; why not just consider this as the round earth?
The definition of a ‘flat’ earth fits what’s described in this model; therefore it is a flat earth model, not round earth, so whatever striking similarities may exist in your conception of a spheroid earth to this are therefore irrelevant.
How far and large are the sun, moon, and stars in this model?
Same distances as the modern mainstream heliocentric model, approximately 93 million miles for the sun, 239,000 miles on average for the moon, and numerous light years for the stars. The sun would then be about 2,713,406 miles in diameter, the moon approximately 6,790 miles in diameter, and the stars will vary with many larger than the sun.
Wait, it seems you got the diameter and circumference mixed up, is that a mistake?
I’m glad you noticed, I used the circumference as the diameter because I’m thinking of them as flat bodies, by traversing them in a straight line, the diameter would be the same as we determine the circumference of them since we treat them as spheres as that’s how they appear from our point of view. However, remember the Ferrari principle, it applies to the celestial bodies as well, they appear as if they are spheres but we are observing a bend in the aether, curved space.
So, the sun, moon, planets, and stars are flat too?
Yes, they all bend aether around them due to their mass.
How does the conspiracy fit here?
As far as I know, this is the only flat earth model that does not have the conspiracy as a necessary consequence. Satellites travel in a straight line according to Newton’s first law while the curved space gives it the orbital path (it’s actually travelling a flat/straight line in curved space) and the supposed pictures of Earth from space and Apollo missions would fall under the Ferrari Effect, the curved space giving the appearance of a round earth. The Apollo Astronauts landed on a flat moon, since if they tried to cross it in a straight/flat line, they would traverse it entirely. However, since the moon has lower mass, the aether bend has a lower magnitude, and therefore lower acceleration on the moon.
Assuming a geocentric framework, wouldn’t stars circling the Earth in a 24-hour period at light-years away need to travel at faster than the speed of light, and isn’t that impossible?
Yes, it may, but according to general relativity, the centrifugal force and “gravitational” field being very large at long distances allows the celestial bodies to move at any speeds, depending on the inertial forces present, if you are going to accept the Earth is at rest relative to the stars with your frame.
“If one
considers the rotating roundabout [earth] as being at rest, the centrifugal gravitational
field assumes enormous values at large distances, and it is consistent with the theory of
General Relativity for the velocities of distant bodies to exceed 3 × 108
m/sec [c] under
these conditions” (An Introduction to the Theory of Relativity, William G. V. Rosser,
1964, p. 460).
« Last Edit: September 24, 2017, 08:11:46 PM by AltSpace »
“Two things are infinite: the universe and human stupidity; and I'm not sure about the universe.”
― Albert Einstein

?

Twerp

  • Gutter Sniper
  • Flat Earth Almost Believer
  • 6540
Re: Davis Relativity Model (Debate/discussion edition)
« Reply #2 on: September 24, 2017, 08:19:22 PM »
Looks good AltSpace. I am working on understanding it but I think some discussion would help me along. Below is a response made by Rabinoz in an earlier thread. I'm sure you've met him by now. He is a hardcore GEer and so will likely approach this from a position of staunchly defending GET. However, he is quite knowledgeable and usually quite reasonable. I hope we can have a good discussion.

I would like to invite you to study this model and tell me what you think?
You mean the  :D model  :D where John Davis deceptively claims Einstein's Relativity Proves The Earth is Flat.

Einstein's General Relativity does not prove that the Earth is Flat and to claim it does, as John Davis, is quite misleading.

Einstein's curved spacetime does not change the shape of the earth or even the apparent shape of the earth at all. The only change is a minute change in diameter (about 1 cm - big deal!).
Quote
Our Local Spacetime
Now that we know what spacetime looks like far from the Earth, we’re ready to explore what it looks like near Earth. Our local spacetime is shown in figure 8.

Figure 8. The shape spacetime near Earth. The red arrow points towards the Earth,
the surface of which is a solid black tube. The blue arrow points into the future.

The lines parallel to the red arrow are lines of constant time, and the lines parallel to the blue arrow are lines of constant distance from the Earth. Notice that the surface of the Earth, the big solid black line, is not a point but a line. This is the worldline of the surface of the Earth. Notice also that the lines scrunch together as you approach the surface of the Earth. This is because lengths and durations are actually shrinking near the Earth. We age slightly slower at sea level than we do on an airplane. (This is related to the gravitational redshift I discussed in an older post.)

If it looks like that scrunching together would eventually lead to the lines of constant distance lying on top of each other, you’re right! If I made the surface of the Earth a smaller and smaller radius, then the lines would eventually lie on top of each other.

The rest in: The Physics Mill, Our Local Spacetime

I have pulled up John Davis a number of times for his deceptive claim the Einstein somehow validates his non-Euclideam model.
Einstein's General Relativity does no such thing and simply does not suggest any "curving" of this nature.

Undoubtedly he does this to give a totally untenable hypothesis a veneer of respectability when in reality it is no more than a thought experment.

It is worth noting that John Davis never really describes his "theory" nor shows what it would do. All he claims is that it "explains everything".
“Heaven is being governed by Devil nowadays..” - Wise

?

Twerp

  • Gutter Sniper
  • Flat Earth Almost Believer
  • 6540
Re: Davis Relativity Model (Debate/discussion edition)
« Reply #3 on: September 24, 2017, 08:31:31 PM »
@OP

Let me ask you this. In your model, does the flat earth have a bottom side? What do you think is there? If we wanted to get there how would we go about it?
“Heaven is being governed by Devil nowadays..” - Wise

*

rabinoz

  • 26528
  • Real Earth Believer
Re: Davis Relativity Model (Debate/discussion edition)
« Reply #4 on: September 24, 2017, 09:19:41 PM »
The Ferrari Effect
The Ferrari Effect is the effect of visualizing a flat plane as being a spheroid due to the curvature of aether (space-time dimensions). This was a prediction made by the philosopher Leo Ferrari that is currently a major concept in the Davis Relativity Model.

You do need to learn a little more about :D Dr. Leo Charles Ferrar and the Ferrari Effect. :D Just watch this and learn all about it.
[youtube][/youtube]
AMAZING! Dr. Leo Charles Ferrari PROVES FLAT EARTH...
See more in AMAZING! Dr. Leo Charles Ferrari PROVES FLAT EARTH, These are clearly men of science, they can't be wrong - right?

*

AltSpace

  • Flat Earth Believer
  • 411
  • Neo-Planarist
Re: Davis Relativity Model (Debate/discussion edition)
« Reply #5 on: September 24, 2017, 09:56:05 PM »
Looks good AltSpace. I am working on understanding it but I think some discussion would help me along. Below is a response made by Rabinoz in an earlier thread. I'm sure you've met him by now. He is a hardcore GEer and so will likely approach this from a position of staunchly defending GET. However, he is quite knowledgeable and usually quite reasonable. I hope we can have a good discussion.

I would like to invite you to study this model and tell me what you think?
You mean the  :D model  :D where John Davis deceptively claims Einstein's Relativity Proves The Earth is Flat.

Einstein's General Relativity does not prove that the Earth is Flat and to claim it does, as John Davis, is quite misleading.

Einstein's curved spacetime does not change the shape of the earth or even the apparent shape of the earth at all. The only change is a minute change in diameter (about 1 cm - big deal!).
Quote
Our Local Spacetime
Now that we know what spacetime looks like far from the Earth, we’re ready to explore what it looks like near Earth. Our local spacetime is shown in figure 8.

Figure 8. The shape spacetime near Earth. The red arrow points towards the Earth,
the surface of which is a solid black tube. The blue arrow points into the future.

The lines parallel to the red arrow are lines of constant time, and the lines parallel to the blue arrow are lines of constant distance from the Earth. Notice that the surface of the Earth, the big solid black line, is not a point but a line. This is the worldline of the surface of the Earth. Notice also that the lines scrunch together as you approach the surface of the Earth. This is because lengths and durations are actually shrinking near the Earth. We age slightly slower at sea level than we do on an airplane. (This is related to the gravitational redshift I discussed in an older post.)

If it looks like that scrunching together would eventually lead to the lines of constant distance lying on top of each other, you’re right! If I made the surface of the Earth a smaller and smaller radius, then the lines would eventually lie on top of each other.

The rest in: The Physics Mill, Our Local Spacetime

I have pulled up John Davis a number of times for his deceptive claim the Einstein somehow validates his non-Euclideam model.
Einstein's General Relativity does no such thing and simply does not suggest any "curving" of this nature.

Undoubtedly he does this to give a totally untenable hypothesis a veneer of respectability when in reality it is no more than a thought experment.

It is worth noting that John Davis never really describes his "theory" nor shows what it would do. All he claims is that it "explains everything".
That link is interesting, however, by reading through it, it seems to contradict how Rabinoz seems to be thinking of it.
I figure it will be up to Rabinoz to elaborate further on this.
Let's go through it:
Quote
As we learned, general relativity tells us that gravity is really a distortion in how we measure distance and duration. In the presence of mass, spacetime distorts so that distances are longer or shorter and time flows more or less quickly. Then objects (under no forces) travel along the straightest possible path through this distorted spacetime. And this motion, which doesn’t look straight, is what we perceive as gravity.
This is a quite significant quote considering what Rabinoz is trying to point out here (claiming that curved space-time in GR gives only a minute change). In GR, the geometry of space is what dictates 'gravity', a straight path through space is significantly distorted from our point of view in the Earth's non-inertial frame of reference with GR.
What is pointed out here is further put into explanation here:
Quote
Of course, because the ball can’t travel faster than light. So a path like that of the red arrow, which is almost a straight line, isn’t valid. The ball has to be within my light cone. Therefore, the worldline of the ball will be some path that travels both forward in time and towards the Earth. And because of the way space and time curve, this will appear as an “accelerating” path.
Demonstrates the magnitude of space-time curvature being defined not by 'consistency' but what I call the 'bend radius' or 'steepness of the space curve'. The constant acceleration to the Earth is consistent across the Earth (ignoring anomalies), this leads to a very important conclusion as signified in the next part.

Quote
I plot the geodesic for the ball in figure 9. Note that it approaches a straight line. That’s because as it accelerates it’s approaching the speed of light (we are neglecting air resistance and exaggerating the distance from the surface of the Earth to make that happen). Note also that the speed of light is a straight line that’s wider than 45 degrees. That’s because of the stretched axis. The path of the ball is curved—it curves with the surface, after all. But it’s as straight as it possibly can be. And that’s what makes it a geodesic.

It’s worth noting that a path away from the Earth would also be a valid worldline. And indeed, it would be just as straight as the path towards the Earth. If, instead of dropping my ball, I threw it upwards at escape velocity, this is indeed the worldline it would choose.
-http://www.thephysicsmill.com/2015/09/06/our-local-spacetime/
As pointed out here, the path of the ball is 'curved' since it curves with the surface (space-time), but is also therefore straight through, labelled as a geodesic. This goes hand in hand with the claim that an object can traverse the Earth in a straight line through curved space-time. Now imagine I threw the ball very high and fast, it could follow a geodesic straight line across Earth, this is orbit, a straight line path through curved space-time (a geodesic). From this, we rightfully define the Earth as flat.
Quote
Let me ask you this. In your model, does the flat earth have a bottom side? What do you think is there? If we wanted to get there how would we go about it?
A 'bottom side' as generally thought of doesn't exist in non-euclidean space-time (aether). With space bending across Earth relative to independent space, a straight line traversal appears to 'circle' it from an external frame of reference. However, this path is defined as 'straight' through curved space-time. With space-time curvature across the Earth, any line drawn straight through Earth will touch the same line traversing the entire Earth. This is what non-euclidean space would produce.
Under the crust, it is molten iron and magma, straight down through it.
« Last Edit: September 24, 2017, 09:58:30 PM by AltSpace »
“Two things are infinite: the universe and human stupidity; and I'm not sure about the universe.”
― Albert Einstein

*

AltSpace

  • Flat Earth Believer
  • 411
  • Neo-Planarist
Re: Davis Relativity Model (Debate/discussion edition)
« Reply #6 on: September 24, 2017, 09:59:30 PM »
The Ferrari Effect
The Ferrari Effect is the effect of visualizing a flat plane as being a spheroid due to the curvature of aether (space-time dimensions). This was a prediction made by the philosopher Leo Ferrari that is currently a major concept in the Davis Relativity Model.

You do need to learn a little more about :D Dr. Leo Charles Ferrar and the Ferrari Effect. :D Just watch this and learn all about it.
[youtube][/youtube]
AMAZING! Dr. Leo Charles Ferrari PROVES FLAT EARTH...
See more in AMAZING! Dr. Leo Charles Ferrari PROVES FLAT EARTH, These are clearly men of science, they can't be wrong - right?
Thanks, not a lot of info on it, so such a video is quite unique. I'll look into it.
“Two things are infinite: the universe and human stupidity; and I'm not sure about the universe.”
― Albert Einstein

*

sandokhan

  • Flat Earth Sultan
  • Flat Earth Scientist
  • 7138
Re: Davis Relativity Model (Debate/discussion edition)
« Reply #7 on: September 24, 2017, 11:37:02 PM »
There is no such thing as the Ferrari effect.

Furthermore, to refer to Einstein again, if his theory of the curvature of space is correct, a square object photographed from space would appear to be circular.

L. Ferrari

There is no such thing as space-time geometry. Here is the step by step demonstration.

Tesla underlined that time was a mere man-made reference used for convenience and as such the idea of a 'curved space-time' was delusional, hence there was no basis for the Relativistic 'space-time' binomium concept.

Motion through space produces the 'illusion of time'.

He considered time as a mere man-made 'measure' of the rate at which events occur such as a distance travelled (in miles or kms) in a certain period of time, for a frame of reference. He considered the 'curving' of space to be absurd (putting it in gentle terms) saying that if a moving body curved space the 'equal and opposite' reaction of space on the body would 'straighten space back out'.

'... Supposing that the bodies act upon the surrounding space causing curving of the same, it appears to my simple mind that the curved spaces must react on the bodies, and producing the opposite effects, straightening out the curves. Since action and reaction are coexistent, it follows that the supposed curvature of space is entirely impossible - But even if it existed it would not explain the motions of the bodies as observed. Only the existence of a field of force can account for the motions of the bodies as observed, and its assumption dispenses with space curvature. All literature on this subject is futile and destined to oblivion. So are all attempts to explain the workings of the universe without recognizing the existence of the ether and the indispensable function it plays in the phenomena.'


G.F. Riemann introduced the additional variables as a supporting theory for his logarithm branch cuts, NOT ever to present time as a new variable.




http://www.maths.tcd.ie/pub/HistMath/People/Riemann/Geom/WKCGeom.html

the abstract concept of n-dimensional geometry to facilitate the geometric representation of functions of a complex variable (especially logarithm branch cut). 'Such researches have become a necessity for many parts of mathematics, e.g., for the treatment of many-valued analytical functions.'

Never did he think to introduce TIME as a separate dimension or variable.

How was this done?

In contrast Riemann’s original non-Euclidian geometry dealt solely with space and was therefore an “amorphous continuum.” Einstein and Minkowski made it metric.

Minkowski's four-dimensional space was transformed by using an imaginary (√-1.ct ) term in place of the real time ( t ). So the coordinates of Minkowski's Four-Dimensional Continuum, ( x1, x2, x3, x4 ) are all treated as space coordinates, but were in fact originally ( x1, x2, x3, t ) or rather ( x1, x2, x3,√-1.ct ), therefore the 4th space dimension x4 is in fact the imaginary √-1.ct substitute. This imaginary 4-dimensional union of time and space was termed by Minkowski as 'world'. Einstein called it 'Spacetime Continuum'. In fact, Minkowski never meant it to be used in curved space. His 4th dimension was meant to be Euclidean dimensions (straight), because it was well before the introduction of General Relativity. Einstein forcibly adopted it for 'curved' or 'None Euclidean' measurements without giving a word of explanations why he could do it. In fact, if there was an explanation Einstein would have given it. Yet, this was how 'Time' became 'Space' or '4th dimensional space' for mathematical purpose, which was then used in 'Spacetime Curvature', 'Ripples of Spacetime' and other applications in General Relativity, relativistic gravitation, which then went on to become Black Hole, etc., ...



EINSTEIN HIMSELF ON THE ABSURDITY OF THE SPACE TIME CONTINUUM CONCEPT:

Einstein, following Minkowski, welded space and time together into what critics have called ‘the monstrosity called space-time’. In this abstract, four-dimensional continuum, time is treated as a negative length, and metres and seconds are added together to obtain one ‘event’. Every point in the spacetime continuum is assigned four coordinates, which, according to Einstein, ‘have not the least direct physical significance’. He says that his field equations, whose derivation requires many pages of abstract mathematical operations, deprive space and time of ‘the last trace of objective reality’.

Total demolition of the theory of relativity:

https://www.theflatearthsociety.org/forum/index.php?topic=30499.msg769750#msg769750

The Walker-Dual superluminal experiment: faster than light travel

https://www.theflatearthsociety.org/forum/index.php?topic=30499.msg1943625#msg1943625




*

JackBlack

  • 21705
Re: Davis Relativity Model (Debate/discussion edition)
« Reply #8 on: September 24, 2017, 11:44:02 PM »
The big issue is that you are appealing to the curvature of space-time, not space.

With your definition, Earth is only flat for specific observers travelling at specific speeds.
For anyone travelling slower, Earth curves upwards (akin to things falling due to gravity), for anyone travelling faster, Earth curves downwards. So for the vast majority of people Earth isn't flat, it curves upwards.

To make it more universal it is better if you can remove this temporal dependence, that way there is just one shape, the shape of Earth in space, and the easiest way to do that is to consider an object travelling at infinite speed, that is an object capable of traversing all of space at a given time, and if you do this, Earth is round. So Earth is round in space, and is just flat for specific objects travelling at specific speeds in specific regions of space-time.

There are other issues as well, for example, while you can simply map all HC observations to GC, that doesn't mean GC is just as good. What GC lacks is an explanation.
For example, why does the sun appear to orbit us and all the other planets orbit the sun?
You can appeal to gravity, but then as Earth is stationary, the curvature of space by the sun should be pulling Earth towards it, especially as it has enough mass to cause the more distant planets to orbit it rather than Earth, and in order to avoid it crashing into the sun it needs to be orbiting the sun.
So that requires Earth to move.

Then there is the issue of how you attempt to explain the Coriolis effect.
When you spin a bucket of water, the water doesn't just sit there. It spins with the bucket (it might take a bit of time to get going). So if the stars were capable of effecting objects on Earth and making them move, then it should be doing the same to Earth, that means Earth wouldn't remain stationary and instead would sync up with the stars and the stars would appear to simply sit there.

And of course, the very distant stars going much much much much much much much faster than the speed of light.
Even Alpha Centuri C (aka Croxima Centuri), at a distance of 4.2 light years, would need to be going at a speed of 26.4 light years a day or roughly 9600 times the speed of light.

So while you can easily map observations to a GC Earth, you can't simply map the explanations. Instead you need to invent new explanations to explain it all.

So the GC part does not work.

*

JackBlack

  • 21705
Re: Davis Relativity Model (Debate/discussion edition)
« Reply #9 on: September 24, 2017, 11:52:36 PM »
Tesla underlined that time was a mere man-made reference used for convenience and as such the idea of a 'curved space-time' was delusional, hence there was no basis for the Relativistic 'space-time' binomium concept.
Time is no more man-made than space.
We make various units to measure it, but it is real, especially as events can happen in a specific region of space by just time passing.

Motion through space produces the 'illusion of time'.
If that was the case then it wouldn't matter how long it took you to run 1 km, it would always seem the same, but we know this isn't the case.

He considered time as a mere man-made 'measure' of the rate at which events occur
i.e. a measure of time.

saying that if a moving body curved space the 'equal and opposite' reaction of space on the body would 'straighten space back out'.
Sure, just like if you curved an object (like a trampoline) the equal and opposite reaction would straighten it back out, but it doesn't.
The equal and opposite reaction doesn't remove the effect.
Also, for the most part, it is an object moving through curved space time that experiences the effect of the curve.
Similar to how fields work, this produces a force on the body that is curving space time.


it appears to my simple mind that the curved spaces must react on the bodies, and producing the opposite effects, straightening out the curves.
No, that would be if space acted on itself.

*

rabinoz

  • 26528
  • Real Earth Believer
Re: Davis Relativity Model (Debate/discussion edition)
« Reply #10 on: September 25, 2017, 01:56:08 AM »
There is no such thing as the Ferrari effect.
:P :P Miracle of miracles! For about the first time I agree with one statement from Sandokhan. :P :P
I doubt it will happen again!

*

Son of Orospu

  • Jura's b*tch and proud of it!
  • Planar Moderator
  • 37834
  • I have artificial intelligence
Re: Davis Relativity Model (Debate/discussion edition)
« Reply #11 on: September 25, 2017, 05:54:08 AM »
There is no such thing as the Ferrari effect.
:P :P Miracle of miracles! For about the first time I agree with one statement from Sandokhan. :P :P
I doubt it will happen again!

I only worry that rab may have received the maximum safe recommended life time dosage of ioning radiation.  Did you even take your potasium pills? 

Re: Davis Relativity Model (Debate/discussion edition)
« Reply #12 on: September 25, 2017, 10:30:55 AM »
I'm sorry Alt, but basically all of this boils down to a round Earth, but using a lot of needless complexity that has no observational support to ultimately describe it as flat.

It would just be easier to admit that the round Earth is the correct functional model rather than jumping through all these hoops hoping to "win" on a technicality.

*

rabinoz

  • 26528
  • Real Earth Believer
Re: Davis Relativity Model (Debate/discussion edition)
« Reply #13 on: September 25, 2017, 07:12:51 PM »
There is no such thing as the Ferrari effect.
:P :P Miracle of miracles! For about the first time I agree with one statement from Sandokhan. :P :P
I doubt it will happen again!

I only worry that rab may have received the maximum safe recommended life time dosage of ioning << spell checker suggests: :D ironing  :D>> radiation.  Did you even take your potasium pills?
What is :P ioning radiation? Do you mean ironing radiation? In case you didn't know there are radiation free irons available.
I have a good supply here and I can forward a couple if you'll just send you to bank account details to rabinoz@radiation_free_ions.co.ng
;D happy to oblige. ;D

Have you been taking "potassium pills"? What an idiot!
Quote
Potassium can cause stomach upset, nausea, diarrhea, vomiting, intestinal gas, and other side effects. Too much potassium is UNSAFE and can cause feelings of burning or tingling, generalized weakness, paralysis, listlessness, dizziness, mental confusion, low blood pressure, irregular heart rhythm, and death.
Those symptoms seem familiar?
I guess that mental confusion you continually suffer from is due to the "potassium pills".
You probably mean "bromide pills".
Quote
Potassium bromide (KBr) is a salt, widely used as an anticonvulsant and a sedative in the late 19th and early 20th centuries, with over-the-counter use extending to 1975 in the US. Its action is due to the bromide ion (sodium bromide is equally effective).
But that's so old fashioned and not usually recommended these days.


But you think that the Ferrari Effect is real? Duh!
Leo Ferrari was a real joker and it looks as though you and John Davis haven't caught on yet.
You did study the video carefully? so that you will understand:
          "light curved by mass makes the earth look a sphere from space" at 14:31 - gee, that sounds like the Ferrari Effect! Would a YouTube video ever mislead you  :D?
          "Earth Moves up and down"  at 19:50 - but I thought that the earth was stationary! Oh, well live and learn :D.
          "Space/non-Space"  at 20:30
          "Cows in Antarctica" at 22:35 - must be real hardy cows, they sure breed 'em tough down there!
       

*

UzZiBiKeR

  • 32
  • Nuclear Reactor Operator
Re: Davis Relativity Model (Debate/discussion edition)
« Reply #14 on: September 25, 2017, 08:36:04 PM »
You folks should know that this site is basically a joke  and you should all try to do something productive in society.
After all the stars go out there will be no light. The end of the stars is approximately 100,000,000,000,000 years. THE UNIVERSE WILL EVENTUALLY GO COMPLETELY DARK!

The End of Everything - Universe Today

End of Stars – 100 trillion years from now
We can look out into the Milky Way and see stars forming all around us. There is still enough remaining gas and dust in the Milky Way to create whole new generations of stars. But when we look at other galaxies, we can see older, elliptical galaxies which have already used up their free gas and dust. Instead of the bright, hot stars we see in star forming regions, these aging red galaxies are cooling down.

One day there won’t be newly forming stars at all. And then one day, the last star will use up the last of its hydrogen fuel, become a red giant and then fade away to a white dwarf. Even the dimmest stars, the cool red dwarfs will use up their fuel – although, it might taken another 10 trillion years or so. They too will turn into black dwarfs.

And so, in about 100 trillion years from now, every star in the Universe, large and small, will be a black dwarf. An inert chunk of matter with the mass of a star, but at the background temperature of the Universe.


The End of Regular Matter – 10^30

years
So now we have a Universe with no stars, only cold black dwarfs. There will also be neutron stars and black holes left over from the time where there were stars in the Universe. The Universe will be completely dark.

A future observer might notice the occasional flash, when some object interacts with a black hole. Its matter will spread out into an accretion disk around the black hole. And for a brief period, it will flare up, emitting radiation. But then it too will be added to the mass of the black hole. And everything will go dark again.

Chunks of matter and binary black dwarfs will merge together creating new black holes, and these black holes will be consumed by even larger black holes. It might be that in the far future, all matter will exist in a few, truly massive black holes.

But even if matter escapes this fate, it’s doomed eventually. Some theories of physics predict that protons are unstable over long periods of time. They just can’t last. Any matter that wasn’t consumed by a black hole will start to decay. The protons will turn into radiation, leaving a fine mist of electrons, positrons, neutrinos and radiation to spread out into space.

Theorists anticipate that all protons in the Universe will decay over the course of 10^30

years.


End of Black Holes – 10^100

Years
Black holes were thought to be one-way streets. Matter goes in, but it doesn’t come out. But famed astrophysicist Stephen Hawking turned that concept on its head with his theory that black holes can evaporate. It’s not much, and it’s not fast, but black holes release a tiny amount of radiation back into space.

As it releases this radiation, the black hole actually loses mass, finally evaporating away entirely. The amount of radiation increases as the black hole loses mass. It’s actually possible that it could generate a final burst of X-rays and gamma rays as it disappears completely. Future observers (who survived their protons decaying) might see the occasional flash in an otherwise dark universe.

And then in about 10^100

years, the last black hole will be gone. All that remains is the radiation emitted.


The End of Everything – 10^100

years and beyond
When the last black hole evaporates, all that will remain in the Universe are photons of radiation, and elementary particles that escaped capture by black holes. The temperature of the entire Universe will reach a final temperature just above absolute zero.

Dark energy may play some future role, continuing the expansion of the Universe, accelerating each of these elementary particles and photons away from each other until they’re effectively cut off from one another. No future gravity will bring them together again.

Perhaps there will be another Big Bang someday. Perhaps the Universe is cyclical and the whole process will start up again.

Perhaps it won’t, and this bleak future of a cold, dead Universe is all that awaits us. It’s not happy, but it’s awe inspiring to consider the long future ahead, and helps us appreciate the vibrant age we live in today.

*

Son of Orospu

  • Jura's b*tch and proud of it!
  • Planar Moderator
  • 37834
  • I have artificial intelligence
Re: Davis Relativity Model (Debate/discussion edition)
« Reply #15 on: September 26, 2017, 06:34:44 AM »
There is no such thing as the Ferrari effect.
:P :P Miracle of miracles! For about the first time I agree with one statement from Sandokhan. :P :P
I doubt it will happen again!

I only worry that rab may have received the maximum safe recommended life time dosage of ioning << spell checker suggests: :D ironing  :D>> radiation.  Did you even take your potasium pills?
What is :P ioning radiation? Do you mean ironing radiation? In case you didn't know there are radiation free irons available.
I have a good supply here and I can forward a couple if you'll just send you to bank account details to rabinoz@radiation_free_ions.co.ng
;D happy to oblige. ;D

Have you been taking "potassium pills"? What an idiot!
Quote
Potassium can cause stomach upset, nausea, diarrhea, vomiting, intestinal gas, and other side effects. Too much potassium is UNSAFE and can cause feelings of burning or tingling, generalized weakness, paralysis, listlessness, dizziness, mental confusion, low blood pressure, irregular heart rhythm, and death.
Those symptoms seem familiar?
I guess that mental confusion you continually suffer from is due to the "potassium pills".
You probably mean "bromide pills".
Quote
Potassium bromide (KBr) is a salt, widely used as an anticonvulsant and a sedative in the late 19th and early 20th centuries, with over-the-counter use extending to 1975 in the US. Its action is due to the bromide ion (sodium bromide is equally effective).
But that's so old fashioned and not usually recommended these days.


But you think that the Ferrari Effect is real? Duh!
Leo Ferrari was a real joker and it looks as though you and John Davis haven't caught on yet.
You did study the video carefully? so that you will understand:
          "light curved by mass makes the earth look a sphere from space" at 14:31 - gee, that sounds like the Ferrari Effect! Would a YouTube video ever mislead you  :D?
          "Earth Moves up and down"  at 19:50 - but I thought that the earth was stationary! Oh, well live and learn :D.
          "Space/non-Space"  at 20:30
          "Cows in Antarctica" at 22:35 - must be real hardy cows, they sure breed 'em tough down there!
       


Lol, typing on a tablet is hard.  OMG, you found the first person who has ever been subjected to the AutoCorrect.  Good catch, where can I put your gold star, special one?

*

rabinoz

  • 26528
  • Real Earth Believer
Re: Davis Relativity Model (Debate/discussion edition)
« Reply #16 on: September 26, 2017, 04:07:30 PM »
Lol, typing on a tablet is hard.  OMG, you found the first person who has ever been subjected to the AutoCorrect.  Good catch, where can I put your gold star, special one?
What do you expect with the smart-aleck remarks you make?

Yes, I often use a tablet, but I can't imagine any autocorrect turning anything into "ioning".
I've tried it on my tablet, Android, on an iPhone 5 and on my computer using Grammarly, which wants it to be "zoning" or "ironing".

Any more excuses?

But just forward the Gold Star to my Nigerian eMail address, but for your own sake don't open any automatic from it.
I have a whole drawer almost full of them under, "jroa's failures".

E&OE Errors & Omissions Expected.

?

Twerp

  • Gutter Sniper
  • Flat Earth Almost Believer
  • 6540
Re: Davis Relativity Model (Debate/discussion edition)
« Reply #17 on: September 26, 2017, 05:41:01 PM »
Is this the best you GEers can do? I'd say this is pretty much a win for FES! Are you ready to concede or can you put up some better arguments?

Just because Ferrari was joking doesn't mean the Ferrari effect isn't happening.
“Heaven is being governed by Devil nowadays..” - Wise

*

AltSpace

  • Flat Earth Believer
  • 411
  • Neo-Planarist
Re: Davis Relativity Model (Debate/discussion edition)
« Reply #18 on: September 26, 2017, 09:19:30 PM »
I'm sorry Alt, but basically all of this boils down to a round Earth,
I addressed this in the FAQ:
Quote
This model seems very similar to the round earth; why not just consider this as the round earth?
The definition of a ‘flat’ earth fits what’s described in this model; therefore it is a flat earth model, not round earth, so whatever striking similarities may exist in your conception of a spheroid earth to this are therefore irrelevant.

Quote
but using a lot of needless complexity that has no observational support to ultimately describe it as flat. It would just be easier to admit that the round Earth is the correct functional model rather than jumping through all these hoops hoping to "win" on a technicality.
But this is correct, at least I am convinced it is with numerous support, and from this, I can conclude a flat earth. I have gave some support. A lot of it has to do with general relativity, which has been experimentally verified over and over, which I have gave some examples.

I don't consider the round earth the default position in which any similarity must support the round earth, hence, why I don't look at it that way.
I could flip it around too, with curved space-time and general relativity, why think of this space-time altering as a round earth rather than flat?
“Two things are infinite: the universe and human stupidity; and I'm not sure about the universe.”
― Albert Einstein

*

AltSpace

  • Flat Earth Believer
  • 411
  • Neo-Planarist
Re: Davis Relativity Model (Debate/discussion edition)
« Reply #19 on: September 26, 2017, 11:20:39 PM »
Thank You for your response Blackjack, I was hoping for feedback from many people. I'll try to elaborate some on your points.

The big issue is that you are appealing to the curvature of space-time, not space.
Well, 4D space-time continuum, as I define 'Aether'.
Quote
With your definition, Earth is only flat for specific observers travelling at specific speeds.
For anyone travelling slower, Earth curves upwards (akin to things falling due to gravity), for anyone travelling faster, Earth curves downwards. So for the vast majority of people Earth isn't flat, it curves upwards.
I'm not sure if I get your point. Maybe I do. I'll try to mimic what you are saying:
All objects traversing Earth will follow lines through space-time, aka 'geodesics', based on it's velocity and distance from Earth. So, if I throw a ball at a 45 degree angle up at 70 mph, it will travel a specific parabolic or hemispheric pattern and hit the ground. That would be a straight line through curved space, as pointed out here:
Quote
Then objects (under no forces) travel along the straightest possible path through this distorted spacetime. And this motion, which doesn’t look straight, is what we perceive as gravity.
-http://www.thephysicsmill.com/2015/09/06/our-local-spacetime/
Now lets look at the definition I used:
Quote
it is something based on space and how it relates to the Earth. A flat plane would be defined by the ability to traverse it in a straight line between two spatial coordinates. A “straight line” would be a line in a constant direction in three dimensional space, or any tangent vector on the surface always touching across it would be a flat surface and therefore a flat earth. So, if the Earth is able to be traversed in a straight line, it follows under the definition of a flat plane. Due to the fact that space is bent around the Earth, a straight line traversing the Earth in bent space will appear to curve relative to an observer in flat space around it, while maintaining a straight line since space curves independent of this line. This means that it maintains itself as straight but in simplistic terms, ‘space curves instead of the line’.
So, basically, a geodesic must be able to traverse the entire Earth through this non-euclidean space. We can define geodesics in terms of paths an object takes in accordance with newton's first law, motion through space-time. Now, lets suppose I threw the baseball very fast, high, and far. Something like at 14,400 mph at 2,000 km above Earth's surface, it then never comes to Earth and constantly 'circles' it. This is orbit. Now, this geodesic through distorted space is straight, and retains a fairly constant altitude above Earth (assuming circular orbit, not elliptical as commonly termed). This follows the definition of straight line traversal and therefore a straight across Earth, as John Davis put it, "We would also like to say that all such traversals are straight or can be seen as straight. It can be said to satisfy this if it can be shown to be parallel to another straight line in space along all such traversals." and "By repeating this experiment again and again with lower apexes of our ball, various orientations, and so on we see the earth itself, not just the paths of satellites, is flat."
A flat surface comes from this geodesic. We can't really derive a curved surface from a non-traversing path as an accurate geometry of Earth.
Quote
To make it more universal it is better if you can remove this temporal dependence, that way there is just one shape, the shape of Earth in space, and the easiest way to do that is to consider an object travelling at infinite speed, that is an object capable of traversing all of space at a given time, and if you do this, Earth is round. So Earth is round in space, and is just flat for specific objects travelling at specific speeds in specific regions of space-time.
If you went to a faster rate, you'd eventually follow a geodesic outside of Earth and leave it, however I'll focus on the traversing the entire earth from point A to point A again. So, as I put it, the entire geodesic crossing Earth through this non-euclidean space as a straight line, is in fact equivalent to traversing the Earth from A to point A again. This would be in accordance with General Relativity, an object travelling through curved space according to newton's first law, a straight path above Earth but an external observer from a point independent of Earth will witness it as appearing to circle Earth. This is the equivalent of someone walking in a straight line on a sidewalk, and two observers, person A and person B, person A walks on the sidewalk in a straight line while Person B watches Person A from a distance. Person A walks in an area with warped space, now, person A will continue to follow a straight line path through space and feel just like that. However, person B will see them walk a curved path, following how space is warping relative to the non-warped space around it, which the observer is in. We can define the entire path of person A as straight, and the only other reference point would be relative to non-warped space around it, but the moment of approach, any traversal will follow warped space in a straight line while being altered relative to space around it.The same applies to Earth with curved space, and with that comes the Ferrari effect, an object in warped space will appear to curve in non-euclidean despite that it is straight in that frame of reference of being within this warped space. From this, we come to the conclusion of a flat earth on the basis of General Relativity.
Quote
There are other issues as well, for example, while you can simply map all HC observations to GC, that doesn't mean GC is just as good. What GC lacks is an explanation.
In General Relativity, absolute motion is not a thing. "Good" is therefore subjective to how you want to interpret and place your reference frame and movements relative to it. So, essentially, Relativity refuted Heliocentrism since any 'centrism' position assumes an absolute motion, of course, the same would apply to Geocentrism and Lunacentrism (Moon centric), etc, but in this case, we can now pick our reference frame and transfer coordinates to a position and relative movements, since in Relativity, being at rest or in motion is only relative to something else. Now, yes, modern mainstream scientists use heliocentrism as an accurate description of our solar system, but that is only because it transfers to a more simplistic coordinate system to use for observation, with geocentrism, you have whirly retograde motions and odd positions that translate to circle/ elliptic paths around a sun. However, in other observations, sometimes geocentrism is assumed, like in mapping the sky in on Earth observations. Other times, studying galaxies can assume a galactic centric frame. In the model I am presenting here, I bring geocentrism in to explain relative motions by using Earth as a center frame.
Quote
For example, why does the sun appear to orbit us and all the other planets orbit the sun?
Relative to us as stationary, these movements give that appearance we can use for heliocentrism as describing these planets and us as orbiting the sun, but with Earth centric stationary frame and coordinates, these objects are moving around us relative to us, using how they relate to the sun as a reasoning against geocentrism would be nill under relativity since that would be if we were assuming absolute could exist or should be assumed.
Quote
You can appeal to gravity, but then as Earth is stationary, the curvature of space by the sun should be pulling Earth towards it, especially as it has enough mass to cause the more distant planets to orbit it rather than Earth, and in order to avoid it crashing into the sun it needs to be orbiting the sun.
So that requires Earth to move.
That is if you assume absolute motion is a thing, but with Relativity, this movement between each other can only be defined as relative with our choice of a frame of reference for coordinate position purposes. So, the Earth being pulled in to the sun would not really be so relevant since relative motions themselves don't exactly determine this.
However, a geocentric explanation to make the other apparent local phenomena work in consistency with a stationary earth would be the Machian mechanics which I brought up and have relied upon, with the stars in this case, working in association with the sun to form a stable equilibrium between Earth by means of geodesic deviation (like stars pulling the sun away in equilibrium to not go into Earth basically), similar to how geosynchronous satellites would be explained as in this particular geocentric frame.
Quote
Then there is the issue of how you attempt to explain the Coriolis effect.
When you spin a bucket of water, the water doesn't just sit there. It spins with the bucket (it might take a bit of time to get going). So if the stars were capable of effecting objects on Earth and making them move, then it should be doing the same to Earth, that means Earth wouldn't remain stationary and instead would sync up with the stars and the stars would appear to simply sit there.
Goes back to what I pointed out earlier. Motion and rest would be define relative to other objects, so, if all objects in the universe are rotating at a rate relative to Earth, a stationary Earth would be derived from this.
Quote
And of course, the very distant stars going much much much much much much much faster than the speed of light.
Even Alpha Centuri C (aka Croxima Centuri), at a distance of 4.2 light years, would need to be going at a speed of 26.4 light years a day or roughly 9600 times the speed of light.
I did respond to this in FAQ:
Quote
Assuming a geocentric framework, wouldn’t stars circling the Earth in a 24-hour period at light-years away need to travel at faster than the speed of light, and isn’t that impossible?
Yes, it may, but according to general relativity, the centrifugal force and “gravitational” field being very large at long distances allows the celestial bodies to move at any speeds, depending on the inertial forces present, if you are going to accept the Earth is at rest relative to the stars with your frame.
“If one
considers the rotating roundabout [earth] as being at rest, the centrifugal gravitational
field assumes enormous values at large distances, and it is consistent with the theory of
General Relativity for the velocities of distant bodies to exceed 3 × 108
m/sec [c] under
these conditions” (An Introduction to the Theory of Relativity, William G. V. Rosser,
1964, p. 460).
So, essentially, in a non-inertial reference frame especially, the speed of light would be a local limit. At star distances with curved space, vectors and relative motions and velocities would need to be derived from the same origin. It isn't a problem.
Quote
So while you can easily map observations to a GC Earth, you can't simply map the explanations. Instead you need to invent new explanations to explain it all.
Eh, you got a point, in that you would need to include mechanics to explain phenomena attributed to Earth's rotation to explain these phenomena (which I did with Mach's principle), but in the case of Relativity, choosing a particular frame of coordinates and a stationary earth isn't an intrinsically incorrect approach and in fact couldn't be.

So, as you can see, this model relies heavily on relativity, hence "Davis Relativity" Model.
“Two things are infinite: the universe and human stupidity; and I'm not sure about the universe.”
― Albert Einstein

*

AltSpace

  • Flat Earth Believer
  • 411
  • Neo-Planarist
Re: Davis Relativity Model (Debate/discussion edition)
« Reply #20 on: September 27, 2017, 01:03:04 AM »
There is no such thing as the Ferrari effect.

Furthermore, to refer to Einstein again, if his theory of the curvature of space is correct, a square object photographed from space would appear to be circular.

L. Ferrari
If it is a defined term that was brought up by John Davis, treating it as an effect with the label 'Ferrari" gives it an existence. Leo Ferrari did bring the concept up, and it was successful with general relativity.

Quote
There is no such thing as space-time geometry. Here is the step by step demonstration.

Tesla underlined that time was a mere man-made reference used for convenience and as such the idea of a 'curved space-time' was delusional, hence there was no basis for the Relativistic 'space-time' binomium concept.
Because Tesla said so debunks all of Relativity and the conceptions of 4D space-time, interesting...
We can measure it, and you admitted to me before that time dilation may occur. So, time is also a variable. It actually is a phenomena after all along with space.

Quote
Motion through space produces the 'illusion of time'.
Time is not an 'illusion', we can measure it, it is a variable, and it varies by movement. Drive 50 km in 20 minutes and someone else drive it in 10 minutes, same motion could be plotted too but distinct time. Time is not a 'mere illusion' just like space isn't. Just like spatial coordinates are not plain illusory, temporal distinctions aren't either.

Quote
He considered time as a mere man-made 'measure' of the rate at which events occur such as a distance travelled (in miles or kms) in a certain period of time, for a frame of reference. He considered the 'curving' of space to be absurd (putting it in gentle terms) saying that if a moving body curved space the 'equal and opposite' reaction of space on the body would 'straighten space back out'.
Faulty logic there. Take a weight, place it on a suspended cloth, it will press into the cloth, that would be the reaction to the weight of the object. It doesn't straighten back out for a reaction, that is the reaction. Now, a big problem here is that it assumes space is like a resistance material which curving has a tendency to react in a flattening out sense. However, that's not how it works, especially not in the model I accept as aether is defined, it is described by the Einstein tensor formula where the presence of energy and mass affects the space-time in which it is in. It does so by warping across large masses. There is no fabric resistance reaction.
Quote
'... Supposing that the bodies act upon the surrounding space causing curving of the same, it appears to my simple mind that the curved spaces must react on the bodies, and producing the opposite effects, straightening out the curves. Since action and reaction are coexistent, it follows that the supposed curvature of space is entirely impossible - But even if it existed it would not explain the motions of the bodies as observed. Only the existence of a field of force can account for the motions of the bodies as observed, and its assumption dispenses with space curvature. All literature on this subject is futile and destined to oblivion. So are all attempts to explain the workings of the universe without recognizing the existence of the ether and the indispensable function it plays in the phenomena.'
No, this misunderstands it entirely, 'opposite' doesn't mean in the definition of opposite terminology, otherwise, the opposite reaction of a rocket taking off would be for it to suddenly stop and not launch since that is the opposite of launching up. However, rather, it is a reaction from the expelling of gas that propels it upwards. In space-time, the presence of mass warping it is an effect of the mass on space-time, there isn't any 'reaction' other than objects following geodesic through those dimensions in accordance with velocity and spatial coordinates, including all parts of the mass itself forming the apparent spherical shape in curved space-time. As defined here, that would be them 'flattening out' actually.

 
Quote
In fact, Minkowski never meant it to be used in curved space. His 4th dimension was meant to be Euclidean dimensions (straight), because it was well before the introduction of General Relativity.
Yes, and I think this is interesting and would serve as the distinction between the round and flat Earth in this 4D space-time continuum defining scenario, one presumes flat space and a round object, and a flat Earth with non-Euclidean space, curved space-time, which is in accordance with general relativity.

Quote
Einstein forcibly adopted it for 'curved' or 'None Euclidean' measurements without giving a word of explanations why he could do it. In fact, if there was an explanation Einstein would have given it. Yet, this was how 'Time' became 'Space' or '4th dimensional space' for mathematical purpose, which was then used in 'Spacetime Curvature', 'Ripples of Spacetime' and other applications in General Relativity, relativistic gravitation, which then went on to become Black Hole, etc., ...
Which included successful predictions BTW, they work well with how our technology AMD discoveries are based. Of course, you kept trying to exclaim any experimentation that fulfills relativity predictions were faked and fitted to all the predictions in dishonest ways, so, it's hard to reason about it then.


Quote
EINSTEIN HIMSELF ON THE ABSURDITY OF THE SPACE TIME CONTINUUM CONCEPT:

Einstein, following Minkowski, welded space and time together into what critics have called ‘the monstrosity called space-time’. In this abstract, four-dimensional continuum, time is treated as a negative length, and metres and seconds are added together to obtain one ‘event’. Every point in the spacetime continuum is assigned four coordinates, which, according to Einstein, ‘have not the least direct physical significance’. He says that his field equations, whose derivation requires many pages of abstract mathematical operations, deprive space and time of ‘the last trace of objective reality’.
But they turned out to have significance, we know time is a distinct variable and something that exists and measurable. This is how events would need to be put into coordinates, space-time as one continuum.
“Two things are infinite: the universe and human stupidity; and I'm not sure about the universe.”
― Albert Einstein

Re: Davis Relativity Model (Debate/discussion edition)
« Reply #21 on: September 27, 2017, 02:51:20 AM »
I'm sorry Alt, but basically all of this boils down to a round Earth,
I addressed this in the FAQ:
Quote
This model seems very similar to the round earth; why not just consider this as the round earth?
The definition of a ‘flat’ earth fits what’s described in this model; therefore it is a flat earth model, not round earth, so whatever striking similarities may exist in your conception of a spheroid earth to this are therefore irrelevant.

Quote
but using a lot of needless complexity that has no observational support to ultimately describe it as flat. It would just be easier to admit that the round Earth is the correct functional model rather than jumping through all these hoops hoping to "win" on a technicality.
But this is correct, at least I am convinced it is with numerous support, and from this, I can conclude a flat earth. I have gave some support. A lot of it has to do with general relativity, which has been experimentally verified over and over, which I have gave some examples.

I don't consider the round earth the default position in which any similarity must support the round earth, hence, why I don't look at it that way.
I could flip it around too, with curved space-time and general relativity, why think of this space-time altering as a round earth rather than flat?
I saw the point in the FAQ, but claiming that similarity between the two is irrelevant doesn't actually make it irrelevant. 

Both models rely heavily on curved space-time and general relativity, but the round Earth model doesn't require extensive and complex, and unobserved, explanations to make it fit as this "flat" Earth model does.

It's kind of like Aristotle's geocentric model is 100% accurate with regards to celestial observation, but requires awkward epicycles that exist for reason other than to make the model work.

As opposed to the heliocentric model which is just as accurate with celestial observation, and is internally consistent with why and how all those things happen.

Your model has a number of "epicycle" type explanations and the round earth model doesn't.  That's the reason for round Earth being better in this case, even if all other things about the models were equal.

*

JackBlack

  • 21705
Re: Davis Relativity Model (Debate/discussion edition)
« Reply #22 on: September 27, 2017, 03:31:36 AM »
All objects traversing Earth will follow lines through space-time, aka 'geodesics', based on it's velocity and distance from Earth. So, if I throw a ball at a 45 degree angle up at 70 mph, it will travel a specific parabolic or hemispheric pattern and hit the ground. That would be a straight line through curved space, as pointed out here
Yes, except I would say a straight path through curved space time.

Now lets look at the definition I used:
So, basically, a geodesic must be able to traverse the entire Earth through this non-euclidean space.
And with that you are pretty much just defining the surface of Earth as flat.
The simple fact is that a geodesic in space-time doesn't need to traverse the entire Earth.
There are infinitely many geodesics which don't come anywhere near Earth, off in space.
There are infinitely many that have the centre of Earth remain at the same distance.
But there are also infinitely many where as you traverse it the surface of Earth gets further and further until you reach a point and then gets closer and closer, or conversely, it gets closer and closer until you reach a point and then gets further and further.

All of these geodesics in space-time are equally valid.
Rejecting all those which don't have Earth be flat is just dishonest special pleading.
You are restricting the applicable geodesics to ones which Earth is flat relative to.
So you are saying Earth is flat, to conclude Earth is flat.

Now, lets suppose I threw the baseball very fast, high, and far.
Instead, compare that to the just as valid and far more common occurrence of throwing it comparatively slowly and at a quite low altitude, like what happens in normal daily life as your prior source describes.
In this case, it still follows what is roughly a geodesic in spacetime (air resistance screws it up). With this, the surface of Earth gets further away until it reaches a peak, and then comes back to the ball.
So for this geodesic, the surface of Earth is curved upwards.

Or compare it to a more common occurrence, an object travelling really really really fast (almost 300 000 000 m/s), a photon of light.
This still follows a geodesic in space time, with Earth now getting closer up until a point and then getting further away. This is a quite extreme case of a hyperbolic orbit.
This geodesic is equally valid but has Earth curve downwards.


"By repeating this experiment again and again with lower apexes of our ball, various orientations, and so on we see the earth itself, not just the paths of satellites, is flat."
No we don't. By repeating this experiment with various parameters, we see that the distance changes based upon speed initial speed, height and angle. In some cases Earth is "flat" in other cases it curves downwards, in others it curves upwards.
Again, excluding those in which it isn't flat is just dishonest special pleading.

This shows we need a more meaningful definition.

The simplest one which allows you to discuss the shape of Earth, is one in which you consider the shape of Earth in space alone, by removing the time aspect and thus consider an object an infinite velocity.


We can't really derive a curved surface from a non-traversing path as an accurate geometry of Earth.
Why not? How is using them any less valid than using your circular orbits?
And what about elliptical orbits, where the surface of Earth would be wavy?

I would say your entire method is fundamentally flawed as you are only looking at the point directly below the orbiting object.
So to really get your traversal (and thus be able to say Earth is flat) you would need a path which goes over every point on Earth, which is fundamentally impossible.

The distance to the other points on the surface of Earth can be determined as well.
This would show that the surface of Earth is always curved away from you. You remaining the same distance away from Earth doesn't mean the surface of Earth isn't curved.
It would appear to be circling you and spinning.
If you say it is impossible to do that, then how do you determine the distance between the surface of Earth and you in the first place?

If you went to a faster rate, you'd eventually follow a geodesic outside of Earth and leave it, however I'll focus on the traversing the entire earth from point A to point A again.
i.e. you will dishonestly restrict your analysis to only contain ones in which Earth appears flat, rejecting all others so you can claim Earth is flat.

We can define the entire path of person A as straight
And it only applies to person A, which means for Earth, it would only apply to the object orbiting Earth, while it is orbiting Earth, not to us while we stand on Earth, and it wouldn't make Earth flat to the person in orbit.

From this, we come to the conclusion of a flat earth on the basis of General Relativity.
No we don't. That is a massive logical leap, which is effectively asserting Earth is flat and space is warped. As I explained, general relativity shows that Earth is round in flat space, and that space time is warped and the path of an object in space time relative to Earth would have Earth appear as getting closer or further away.

In General Relativity, absolute motion is not a thing.
Which means saying Earth is absolutely still (as in geocentrism) makes no sense.
However, it doesn't make motion not a thing. Instead it has non-inertial, i.e. accelerating reference frames.
It has space-time warp, and mass causes this warping of space time, which results in objects accelerating towards objects with mass. This means that they will move. This means a stationary Earth makes no sense.

With GR everything is in motion.

So, essentially, Relativity refuted Heliocentrism, as hel since any 'centrism' position assumes an absolute motion
No it doesn't.
Heliocentrism in the modern sense has the sun being approximately the centre of the solar system It does not discuss the motion of the solar system relative to anything else, and it doesn't even have the sun fixed. As such, the entire solar system can be in motion.
What is important is that unlike other centred positions, like geocentrism, it can explain the observed motions of objects.

we can now pick our reference frame and transfer coordinates to a position and relative movements
Again, while this maps the movements, it doesn't map the explanation.
With GC you need an explanation for why Earth remains still, the sun orbits us and the planets orbit the sun.

that is only because it transfers to a more simplistic coordinate system to use for observation
No, it is because it has explanatory power.
With GR, every mass attracts every other mass.
That means if the sun is orbiting Earth, Earth is orbiting the sun. Due to the much larger mass of the sun, that puts the barycentre (the point they are both orbiting) in the sun. Similarly, all other planets orbit the sun. This is why it is used. It has explanatory power and thus can explain the position of objects.

However, in other observations, sometimes geocentrism is assumed
No it isn't. HC is used and it is translated to a GC frame.

Quote
For example, why does the sun appear to orbit us and all the other planets orbit the sun?
Relative to us as stationary, these movements give that appearance we can use for heliocentrism as describing these planets and us as orbiting the sun, but with Earth centric stationary frame and coordinates, these objects are moving around us relative to us, using how they relate to the sun as a reasoning against geocentrism would be nill under relativity since that would be if we were assuming absolute could exist or should be assumed.
None of this addressed the question I raised.
Again, HC does not assume absolute motion. It assumes acceleration caused by gravity. This explains the path of the objects in the solar system in a HC model. The GC model cannot explain it.

So I ask again: for the GC system, why does the sun orbit us and all the other planets orbit the sun?

(I'm just going to skip the rest of the repetition of your strawmen and just focus on new points I see.)

would be the Machian mechanics which I brought up and have relied upon
You mean the ones I showed don't work?

Yes, it may, but according to general relativity, the centrifugal force and “gravitational” field being very large at long distances allows the celestial bodies to move at any speeds
And this shows you have no idea what you are talking about. How does centrifugal and gravitational forces allow them to move at any speed?

if you are going to accept the Earth is at rest relative to the stars with your frame.
So if you accept that Earth is stationary, then you accept anything to make it so?

So, essentially, in a non-inertial reference frame especially, the speed of light would be a local limit
Ahh, the good old local limit, where observations outside it make no sense, as the local limit is the limit of the frame of reference. Beyond this local limit the frame of reference tears itself apart.

Re: Davis Relativity Model (Debate/discussion edition)
« Reply #23 on: September 27, 2017, 12:52:12 PM »
All objects traversing Earth will follow lines through space-time, aka 'geodesics', based on it's velocity and distance from Earth. So, if I throw a ball at a 45 degree angle up at 70 mph, it will travel a specific parabolic or hemispheric pattern and hit the ground. That would be a straight line through curved space, as pointed out here:
Quote
Then objects (under no forces) travel along the straightest possible path through this distorted spacetime. And this motion, which doesn’t look straight, is what we perceive as gravity.
-http://www.thephysicsmill.com/2015/09/06/our-local-spacetime/
Now lets look at the definition I used:
Quote
it is something based on space and how it relates to the Earth. A flat plane would be defined by the ability to traverse it in a straight line between two spatial coordinates. A “straight line” would be a line in a constant direction in three dimensional space, or any tangent vector on the surface always touching across it would be a flat surface and therefore a flat earth. So, if the Earth is able to be traversed in a straight line, it follows under the definition of a flat plane. Due to the fact that space is bent around the Earth, a straight line traversing the Earth in bent space will appear to curve relative to an observer in flat space around it, while maintaining a straight line since space curves independent of this line. This means that it maintains itself as straight but in simplistic terms, ‘space curves instead of the line’.
So, basically, a geodesic must be able to traverse the entire Earth through this non-euclidean space. We can define geodesics in terms of paths an object takes in accordance with newton's first law, motion through space-time. Now, lets suppose I threw the baseball very fast, high, and far. Something like at 14,400 mph at 2,000 km above Earth's surface, it then never comes to Earth and constantly 'circles' it. This is orbit. Now, this geodesic through distorted space is straight, and retains a fairly constant altitude above Earth (assuming circular orbit, not elliptical as commonly termed). This follows the definition of straight line traversal and therefore a straight across Earth, as John Davis put it, "We would also like to say that all such traversals are straight or can be seen as straight. It can be said to satisfy this if it can be shown to be parallel to another straight line in space along all such traversals." and "By repeating this experiment again and again with lower apexes of our ball, various orientations, and so on we see the earth itself, not just the paths of satellites, is flat."

Firstly, it's important to distinguish between spacetime and space. A free-falling object will follow a 'straight' path through curved spacetime, but not, in general, through curved space (unless travelling at light-speed).

Secondly, you seem to be defining 'flat' as meaning 'the surface of a body that can be orbited'. I don't see your description being more restrictive than this. And I don't see this definition being justified.

If I read your justification correctly, it seems to relate to the fact that the surface remains spatially equidistant from a satellite in circular orbit. However the satellite's orbit is not straight in curved space, but in curved spacetime, as noted above. Further, the same logic applied to an elliptical orbit would imply an undulating earth.
« Last Edit: September 27, 2017, 01:00:13 PM by Copper Knickers »

?

Master_Evar

  • 3381
  • Well rounded character
Re: Davis Relativity Model (Debate/discussion edition)
« Reply #24 on: September 30, 2017, 02:44:29 AM »
On Boots request, I present a short part of a video that presents a similar "flat space" idea:
" class="bbc_link" target="_blank" rel="noopener noreferrer">
(TL:DW, our universe might be a "Hologram" painted onto the (flat) event horizon of a black hole).
So, you might not be that far off Davis, even though the math and specific phenomena behind it might be wrong.
Math is the language of the universe.

The inability to explain something is not proof of something else.

We don't speak for reality - we only observe it. An observation can have any cause, but it is still no more than just an observation.

When in doubt; sources!

?

Twerp

  • Gutter Sniper
  • Flat Earth Almost Believer
  • 6540
Re: Davis Relativity Model (Debate/discussion edition)
« Reply #25 on: October 01, 2017, 09:58:31 AM »
RE the Davis Relativity Model:

Quote from: Master_Evar
Well, it hasn't been debunked. The idea that "space is flat" is kind of impossible to either prove or disapprove. However, it's way more complicated than simply bending space according to general relativity as Davis suggests. If I had enough time, I think I could prove the complexity of the idea with closed knots and things such as möbius strips (combined with maths).

The biggest problem is that you can't just bend 2-dimensional space and get a 3-dimensional figure. However, you can bend a 2-dimensional figure in 3-dimensional space and get a 3-dimensional appearance. Take a circular bit of rope, then twist it in the middle. You can do this without having to pass the rope through itself, but only because one of the rope-parts is on top of the other. However, if you have a circle on a plane, there is no way to twist it to give the appearance of an 8. Things such as möbius strips ONLY work in 3 dimensions, no matter how much you bend the space it is located in. Just like a klein bottle is impossible to produce in 3 dimensions, but works fine in 4 dimensions.

Quote from: Master_Evar
I'll try to look for some sources that better explains it and refine my thoughts.
“Heaven is being governed by Devil nowadays..” - Wise

*

AltSpace

  • Flat Earth Believer
  • 411
  • Neo-Planarist
Re: Davis Relativity Model (Debate/discussion edition)
« Reply #26 on: October 01, 2017, 10:45:43 PM »
I saw the point in the FAQ, but claiming that similarity between the two is irrelevant doesn't actually make it irrelevant.
It kinda does when you have all this reasoning behind it and someone thinks they can ignore it without addressing it. 

Quote
Both models rely heavily on curved space-time and general relativity,
That statement is challenged here by stating that it is curved space-time and general relativity that demonstrates a flat earth and the round earth relies on Minkowski space-time and Newtonian Gravitation representations.
Quote
but the round Earth model doesn't require extensive and complex, and unobserved, explanations to make it fit as this "flat" Earth model does.
You start off by saying that the round earth relies heavily on curved space-time and GR and then claim my use of it is too extensive and full of unobserved explanations? Odd indeed. GR implies the Earth is flat by definition, what I'm saying here isn't new or some radical reinterpretation of it, it is derived from it that geodesics traverse Earth, a straight line through curved space-time.

Quote
It's kind of like Aristotle's geocentric model is 100% accurate with regards to celestial observation, but requires awkward epicycles that exist for reason other than to make the model work.

As opposed to the heliocentric model which is just as accurate with celestial observation, and is internally consistent with why and how all those things happen.

Your model has a number of "epicycle" type explanations and the round earth model doesn't.  That's the reason for round Earth being better in this case, even if all other things about the models were equal.
You are right, I should change it like what I was previously thinking to keep this misunderstanding out of it, geocentricity and heliocentricity doesn't exist outside of personal conception, I should stress that more and explain it as acentric (relative motion through aether).
“Two things are infinite: the universe and human stupidity; and I'm not sure about the universe.”
― Albert Einstein

*

JackBlack

  • 21705
Re: Davis Relativity Model (Debate/discussion edition)
« Reply #27 on: October 01, 2017, 11:16:12 PM »
That statement is challenged here by stating that it is curved space-time and general relativity that demonstrates a flat earth and the round earth relies on Minkowski space-time and Newtonian Gravitation representations.
And that statement has also been challenged.
The curved space-time and GR do not demonstrate a FE, nor does the RE rely upon minkowski space-time or newtonian gravity.

GR implies the Earth is flat by definition
No it doesn't, and I have explained why.
Firstly, it would only be true for specific orbits, those which are perfectly circular. Any other velocity/distance, and then no more FE.
Even in these orbits, the only way you can say Earth is flat is that you remain the same distance away from the point on the surface below you. It completely ignores the fact that the distance to other points do not match that for a flat surface, or the fact that at best you get a line around Earth, not the entire surface.
But more importantly, the reference frames of GR are inherently local.
Just because the straight line is there, doesn't mean it will still be straight some distance away.

So no, GR does not indicate Earth is flat.

it is derived from it that geodesics traverse Earth, a straight line through curved space-time
And ignoring all the ones in which Earth is not flat, and ignoring several other problems.

*

AltSpace

  • Flat Earth Believer
  • 411
  • Neo-Planarist
Re: Davis Relativity Model (Debate/discussion edition)
« Reply #28 on: October 02, 2017, 12:08:16 AM »
Yes, except I would say a straight path through curved space time.
That works.
The other points boil down to this:
Quote
No we don't. By repeating this experiment with various parameters, we see that the distance changes based upon speed initial speed, height and angle. In some cases Earth is "flat" in other cases it curves downwards, in others it curves upwards.
Again, excluding those in which it isn't flat is just dishonest special pleading.
Did you not read the definition I provided? Here it is:
Quote
A flat plane would be defined by the ability to traverse it in a straight line between two spatial coordinates. A “straight line” would be a line in a constant direction in three dimensional space, or any tangent vector on the surface always touching across it would be a flat surface and therefore a flat earth. So, if the Earth is able to be traversed in a straight line, it follows under the definition of a flat plane.
I am taking the geodesics traversing the Earth's surface because that is what defines it in this case, and as you have agreed, it's flat. I'll simply ignore your points on non-traversing geodesics as they purposely take this all out of context.
Quote
The simplest one which allows you to discuss the shape of Earth, is one in which you consider the shape of Earth in space alone, by removing the time aspect and thus consider an object an infinite velocity.
Which you actually can't do. Space and time are wrapped into the aether concept here. However, this is rendered incorrect due to the fact that geodesics traversing the Earth are straight.

Quote
And it only applies to person A, which means for Earth, it would only apply to the object orbiting Earth, while it is orbiting Earth, not to us while we stand on Earth, and it wouldn't make Earth flat to the person in orbit.
Traversals geodesics are straight regardless of where you are standing on Earth.

Quote
As I explained, general relativity shows that Earth is round in flat space, and that space time is warped

Is this some sort of joke?
Quote
Which means saying Earth is absolutely still (as in geocentrism) makes no sense.
I thought about it and I think I'll change it to a relative motion acentric explanation to keep the absolutist ways of thinking about it type of misunderstandings out.
Quote
However, it doesn't make motion not a thing.

It means motion is relative, and you saying that has me thinking you are probably missing the point.
Quote
It does not discuss the motion of the solar system relative to anything else, and it doesn't even have the sun fixed. As such, the entire solar system can be in motion.
Yep, you are missing the point. The motion is relative, including the entire solar system, heliocentrism assumes all objects in the solar system are rotating around the stationary sun (not considering it's relative motion outside of the system of course), that implies objective motion.
Quote
What is important is that unlike other centred positions, like geocentrism, it can explain the observed motions of objects.
All can, relative to that stationary frame, we just say, 'it moves this way relative to it'.

Quote
No, it is because it has explanatory power.
Not in relativity, it doesn't exist, it is just conceptual for simplification purposes of analyzing relative motions.
Quote
That means if the sun is orbiting Earth, Earth is orbiting the sun. Due to the much larger mass of the sun, that puts the barycentre (the point they are both orbiting) in the sun. Similarly, all other planets orbit the sun. This is why it is used. It has explanatory power and thus can explain the position of objects.
i.e. assuming objective motion.

Quote
No it isn't. HC is used and it is translated to a GC frame.
That is the same thing as assuming geocentrism, at least in how I'm using it.

Quote
Again, HC does not assume absolute motion.

Wikipedia:
Quote
Heliocentrism[1] is the astronomical model in which the Earth and planets revolve around the Sun at the center of the Solar System.
-https://en.wikipedia.org/wiki/Heliocentrism
If that isn't 'absolute', then I don't know what is.

Quote
So I ask again: for the GC system, why does the sun orbit us and all the other planets orbit the sun?
As I said already, I'll backtrack now since I feel it seems to not be accurately portraying the big point with my model.
Explaining a bit:
Motion is relative between objects through the aether, motion is defined relative to frames of reference. The observations we derive in our frame relating to motion are relative between frames of reference, if none other exist, you can only be stationary.
This becomes an acentric relativist explanation of the solar system and universe in the absolute sense.

Quote
You mean the ones I showed don't work?
You didn't address it's validity, so I don't know what you are talking about here.

Quote
And this shows you have no idea what you are talking about. How does centrifugal and gravitational forces allow them to move at any speed?
Because c (speed of light as measured in any inertial reference frame) doesn't apply to non-inertial frames, which includes centrifugal and 'G forces'.
Quote
Ahh, the good old local limit, where observations outside it make no sense, as the local limit is the limit of the frame of reference. Beyond this local limit the frame of reference tears itself apart.
You can't meaningfully compare velocities of distant objects directly or compare vectors from distinct points without bringing them to a same origin, our frame of reference.
By choosing coordinates, you can assume the speed of light that is distant from you as at any value.
“Two things are infinite: the universe and human stupidity; and I'm not sure about the universe.”
― Albert Einstein

*

AltSpace

  • Flat Earth Believer
  • 411
  • Neo-Planarist
Re: Davis Relativity Model (Debate/discussion edition)
« Reply #29 on: October 02, 2017, 12:15:47 AM »
Firstly, it's important to distinguish between spacetime and space. A free-falling object will follow a 'straight' path through curved spacetime, but not, in general, through curved space (unless travelling at light-speed).

Secondly, you seem to be defining 'flat' as meaning 'the surface of a body that can be orbited'. I don't see your description being more restrictive than this. And I don't see this definition being justified.

If I read your justification correctly, it seems to relate to the fact that the surface remains spatially equidistant from a satellite in circular orbit. However the satellite's orbit is not straight in curved space, but in curved spacetime, as noted above. Further, the same logic applied to an elliptical orbit would imply an undulating earth.
As I pointed out, 4D space-time is wrapped into a continuum, an 'aether'.
I don't see the problem you are pointing out here, so I'll assume you aren't.
Thanks for the comment.
“Two things are infinite: the universe and human stupidity; and I'm not sure about the universe.”
― Albert Einstein